Finding inverse in polynomial factor ring

Click For Summary
To find the inverse of r in the polynomial factor ring R = F[x]/<h>, where r = 1 + t - t^2 and F = Z_7, the setup involves multiplying an element of R by r to equal 1. The polynomial h has a degree of 3, leading to the structure R = {a + bt + ct^2}. The user attempted to solve by expanding the product and substituting for t^3, but encountered issues with the coefficients, particularly getting a constant coefficient of zero. The discussion suggests using the Extended Euclidean Algorithm to find the inverse, highlighting the potential for calculation errors in this tedious process.
PsychonautQQ
Messages
781
Reaction score
10

Homework Statement


find the inverse of r in R = F[x]/<h>.
r = 1 + t - t^2
F = Z_7 (integers modulo 7), h = x^3 + x^2 -1

Homework Equations


None

The Attempt at a Solution


The polynomial on bottom is of degree 3, so R will look like:
R = {a + bt + ct^2 | a,b,c are elements of z_7 and x^3 = 1 - ^2}

To solve this problem I realized that the inverse must obviously have the form of some element in R, so I set up:
(a + bt + ct^2)(1 + t - t^2) = 1

then I multiplied it all out whilst continuously substituting for t^3 and then solving for coefficients where the constant coefficient should equal 1 and the other two should equal 0.

I did all of this and got the constant coefficient to be zero and nonzero answers for the other two >.<. I checked my calculations and can't find an error (doesn't necessarily mean there isn't one...), is something wrong with the way I set up the problem? is my substitution for x^3 correct?
 
Physics news on Phys.org
PsychonautQQ said:

Homework Statement


find the inverse of r in R = F[x]/<h>.
r = 1 + t - t^2
F = Z_7 (integers modulo 7), h = x^3 + x^2 -1

Homework Equations


None

The Attempt at a Solution


The polynomial on bottom is of degree 3, so R will look like:
R = {a + bt + ct^2 | a,b,c are elements of z_7 and x^3 = 1 - ^2}

To solve this problem I realized that the inverse must obviously have the form of some element in R, so I set up:
(a + bt + ct^2)(1 + t - t^2) = 1

then I multiplied it all out whilst continuously substituting for t^3 and then solving for coefficients where the constant coefficient should equal 1 and the other two should equal 0.

I did all of this and got the constant coefficient to be zero and nonzero answers for the other two >.<. I checked my calculations and can't find an error (doesn't necessarily mean there isn't one...), is something wrong with the way I set up the problem? is my substitution for x^3 correct?

The usual way to do this is to use the Extended Euclidean Algorithm to explicitly write Bezout's identity. gcd(h,r)=a*h+b*r. Then mod both sides by h. Does that sound familiar? It is kind of a tedious calculation and it's easy to make a mistake. What did you do with t^4?
 
Last edited:
Question: A clock's minute hand has length 4 and its hour hand has length 3. What is the distance between the tips at the moment when it is increasing most rapidly?(Putnam Exam Question) Answer: Making assumption that both the hands moves at constant angular velocities, the answer is ## \sqrt{7} .## But don't you think this assumption is somewhat doubtful and wrong?

Similar threads

  • · Replies 3 ·
Replies
3
Views
2K
  • · Replies 1 ·
Replies
1
Views
2K
  • · Replies 13 ·
Replies
13
Views
3K
Replies
9
Views
2K
  • · Replies 18 ·
Replies
18
Views
5K
  • · Replies 4 ·
Replies
4
Views
1K
  • · Replies 12 ·
Replies
12
Views
3K
  • · Replies 12 ·
Replies
12
Views
4K
  • · Replies 3 ·
Replies
3
Views
2K
  • · Replies 10 ·
Replies
10
Views
2K